Suppose that the UV light of wavelength 250 nm has an intensity of 20 mW cm2. If the emitted electrons are collected by applying a positive bias to the opposite electrode, what will be the photoelectric current density?

Answers

Answer 1

To find the photoelectric current density, we need the area of the electrode. Without the value of the area, we cannot calculate the current density.

To calculate the photoelectric current density, we need to use the equation for photoelectric current:

I = q * Φ * A

where I is the current, q is the charge of an electron (1.6 x 10^-19 C), Φ is the number of photoelectrons emitted per unit area per unit time (also known as the photoelectric emission rate), and A is the area of the electrode.

The photoelectric emission rate depends on the intensity of light and the efficiency of the photoelectric effect. In this case, we assume that all incident photons with a wavelength of 250 nm are absorbed and result in the emission of one photoelectron.

Given:

Wavelength of light, λ = 250 nm = 250 x 10^-9 m

Intensity of light, I = 20 mW/cm^2 = 20 x 10^-3 W/m^2

Charge of an electron, q = 1.6 x 10^-19 C

Area of the electrode, A (not given)

Know more about photoelectric current density here:

https://brainly.com/question/28285184

#SPJ11


Related Questions

Evidence suggests that discovery learning is not effective in improving observer's performance.
T/F

Answers

T (True). Evidence suggests that discovery learning is not effective in improving observer's performance.

Evidence from research studies indicates that discovery learning, where learners explore and discover concepts or solutions on their own, may not be as effective in improving observer's performance compared to other instructional methods. Some studies have found that guided instruction, which provides explicit guidance and support, leads to better learning outcomes and performance than discovery learning alone. Guided instruction helps learners acquire foundational knowledge and skills before engaging in more independent exploration. While discovery learning can have benefits in certain contexts and for specific learning objectives, research suggests that a balanced approach combining both guided instruction and opportunities for independent exploration may be more effective in promoting learning and improving performance.

Know more about research studies here:

https://brainly.com/question/28487173

#SPJ11

a 503×10−6 f capacitor is discharged through a resistor, whereby its potential difference decreases from its initial value of 94.5 v to 16.3 v in 3.21 s . find the resistance of the resistor.

Answers

The resistance of the resistor is approximately 259.4 ohms.

To find the resistance of the resistor, we can use the formula for the discharge of a capacitor in an RC circuit:

V(t) = V0 * e^(-t/RC)

Where:

V(t) is the potential difference at time t

V0 is the initial potential difference

t is the time

R is the resistance

C is the capacitance

We are given:

V0 = 94.5 V (initial potential difference)

V(t) = 16.3 V (potential difference after time t)

t = 3.21 s (time)

C = 503×10^(-6) F (capacitance)

Plugging in the values, we get:

16.3 = 94.5 * e^(-3.21/(R * 503×10^(-6)))

To find the resistance, we need to solve this equation for R. Rearranging the equation, we have:

e^(-3.21/(R * 503×10^(-6))) = 16.3 / 94.5

Taking the natural logarithm (ln) of both sides, we get:

-3.21/(R * 503×10^(-6)) = ln(16.3 / 94.5)

Now, we can solve for R by isolating it:

R = -3.21 / (ln(16.3 / 94.5) * 503×10^(-6))

Calculating the right side of the equation, we find:

R ≈ 259.4 Ω

Know more about resistance here:

https://brainly.com/question/32301085

#SPJ11

explain the following statement: a transaction is a logical unit of work. give an example not in the book and explain it.

Answers

In a database management system, a transaction is a logical unit of work.

A transaction is a group of operations that must all succeed or all fail; the database system is in an undefined state if any of the operations fail. Transactions provide reliable consistency for database updates. However, a logical unit of work is a portion of a program that is treated as a single logical unit from the point of view of data updates or queries. The logical unit of work is used in conjunction with transaction management systems to ensure that data updates occur as a single logical unit of work and can be rolled back if any part of the logical unit of work fails. As a result, a transaction is a logical unit of work.

Example: Let's pretend that a student is enrolling in courses. A single transaction would be the entire enrollment process, from logging into the system to selecting courses to registering for them. If any portion of the transaction fails, the entire enrollment process will fail.

Learn more about the logical unit of work at:

brainly.com/question/32344402

#SPJ11

data for a kaplan turbine is given as follows; head=5m & flow rate of water =120m³/s; runner speed=120rev/min; runner diameter =5m; hub tip ratio of the runner=0.4; runner blade angle of entry =60⁰; overall efficiency =80%. exit from the runner is axial. calculate the; a power b)specific speed c) exit angle of the inlet guide vanes d) exit angle of runner blades; and e)hydraulic efficiency

Answers

a) Power:

The power output of the turbine can be calculated using the following formula:

Power = (Head × Flow rate × Gravity) / Overall efficiency

Where gravity is the acceleration due to gravity (approximately 9.81 m/s²).

Substituting the given values:

Power = (5 m × 120 m³/s × 9.81 m/s²) / 0.8

Power ≈ 9112.5 kW

b) Specific speed:

Specific speed (Ns) is a dimensionless parameter used to characterize the performance of a turbine. It can be calculated using the following formula:

Ns = (N √Q) / H^(3/4)

Where N is the runner speed in revolutions per minute, Q is the flow rate in cubic meters per second, and H is the head in meters.

Substituting the given values:

Ns = (120 rev/min √120 m³/s) / (5 m)^(3/4)

Ns ≈ 774.12

c) Exit angle of the inlet guide vanes:

The exit angle of the inlet guide vanes can be calculated using the following formula:

Exit angle = Blade angle of entry - Hub tip ratio × 90°

Substituting the given values:

Exit angle = 60° - 0.4 × 90°

Exit angle ≈ 24°

d) Exit angle of runner blades:

The exit angle of the runner blades in an axial flow turbine is approximately equal to the exit angle of the inlet guide vanes. Therefore, the exit angle of the runner blades would also be approximately 24°.

e) Hydraulic efficiency:

Hydraulic efficiency (ηh) can be calculated using the following formula:

ηh = (Head × Flow rate × Gravity × Overall efficiency) / (Power / 1000)

Substituting the given values:

ηh = (5 m × 120 m³/s × 9.81 m/s² × 0.8) / (9112.5 kW / 1000)

ηh ≈ 0.537 or 53.7%

To summarize:

a) Power ≈ 9112.5 kW

b) Specific speed ≈ 774.12

c) Exit angle of the inlet guide vanes ≈ 24°

d) Exit angle of runner blades ≈ 24°

e) Hydraulic efficiency ≈ 53.7%

1. Copper-Rich copper-beryllium alloys are precipitation hardenable. After consulting the portion of the phase diagram, do the following.
a) Specify the range of compositions over which these alloys may be precipitation hardened and
b) briefly describe the heat-treatment process (in terms of temperature) that would be used to precipitation harden an alloy having a composition of your choosing, yet lying within the range given for part a.

Answers

Copper-rich copper-beryllium alloys are precipitation hardenable. They have compositions ranging from 0.2 to 2 wt percent beryllium, with copper making up the remaining percentage. The alloys have a high electrical conductivity and are used in the electronics industry.

The following steps can be used to heat-treat an alloy of your choosing that lies within the composition range specified in part a.The range of compositions for which copper-rich copper-beryllium alloys can be precipitation hardened is from 0.2 to 2 wt percent beryllium, with the rest being copper. They are widely used in the electronics industry due to their high electrical conductivity. The heat-treatment process can be briefly described as follows: Step 1: Heating the alloy to a temperature range of 315°C to 425°C for 1 to 4 hours. Step 2: Allow the alloy to cool to room temperature or near it. Step 3: Age the alloy at a temperature range of 160°C to 315°C for several hours. The alloy is then removed from the furnace and allowed to cool to room temperature or near it. The result of this heat treatment is the precipitation of an intermetallic compound called CuBe2. This is the cause of the hardening of the alloy.

To know more about hardenable visit:

https://brainly.com/question/31116300

#SPJ11

.Consider the following recursive definition of a set S of strings. 1. Any letter in {a,b,c) is in S; 2. If x ∈ S, then xx ∈ S: 3. If x ∈ s, then cx ∈ S Which of the following strings are in S? A) ba B) a C) ca D) cbca E) acac F) X G) cb H) cbcb I) cba J) cbccbc K) aa L) ccbccb M) ccaca N) Occb

Answers

The following strings belong to the given set S:

ba, a, ca, cbca, acac, cbcb, cba, cbccbc, aa, ccbccb, ccaca, and Occb.

Explanation:

According to the given recursive definition of a set S of strings, any letter in {a,b,c) is in S, If x ∈ S, then xx ∈ S, and if x ∈ S, then cx ∈ S. Here are the strings belonging to the given set S:

ba (since b and a both belong to {a,b,c})

a (since a belongs to {a,b,c})

ca (since a belongs to {a,b,c}, and c added to a belongs to S)

cbca (since a and c both belong to {a,b,c} and adding them in the order c, b, c, a, respectively belongs to S)

acac (since a and c both belong to {a,b,c} and adding them in the order a, c, a, c respectively belongs to S)

cbcb (since b and c both belong to {a,b,c} and adding them in the order c, b, c, b respectively belongs to S)

cba (since a and c both belong to {a,b,c} and adding them in the order c, b, a respectively belongs to S)

cbccbc (since b and c both belong to {a,b,c} and adding them in the order c, b, c, c, b, c respectively belongs to S)

aa (since a belongs to {a,b,c} and adding a to itself belongs to S)

ccbccb (since b and c both belong to {a,b,c} and adding them in the order c, c, b, c, c, b respectively belongs to S)

ccaca (since a and c both belong to {a,b,c} and adding them in the order c, c, a, c, a respectively belongs to S)

Occb (since b and c both belong to {a,b,c} and adding them in the order c, b, c, c, b, O respectively belongs to S)

Hence, the strings belonging to the given set S are ba, a, ca, cbca, acac, cbcb, cba, cbccbc, aa, ccbccb, ccaca, and Occb.

Learn more about strings here:

https://brainly.com/question/30765679

3SPJ11

Use a clock, a counter, and a demultiplexer to control whether a stop light is red, green or yellow. The stop light should be green for 40 seconds, followed by yellow for 10 seconds, and then red for 30 seconds, and then repeat. You can simply write by each output of the demultiplexer one of the words green, yellow, or red. Don't forget to specify how many Hz the clock needs to be.

Answers

A circuit with one input and numerous outputs is called a demultiplexer, or dmux for short.

Thus, When a circuit wants to convey a signal to a number of different devices, it uses it.  This sounds a lot like the description of a decoder, but a decoder sends a signal among a small number of devices, whereas a demultiplexer sends a signal among a large number of devices and circuit.

Demultiplexers are utilized frequently enough to have their own schematic symbol. It is possible to expand this circuit in two different ways.

Either the number of inputs that are passed through or the number of signals that are conveyed can be increased. All that is necessary to increase the number of inputs that can be processed is a larger line decoder and circuit.

Thus, A circuit with one input and numerous outputs is called a demultiplexer, or dmux for short.

Learn more about Circuit, refer to the link:

https://brainly.com/question/12608516

#SPJ4

water flows in a cast-iron pipe of 550-mm diameter at a rate of 0.10 m3/s. determine the friction factor for this flow.

Answers

For water flowing in a cast-iron pipe of 550 mm diameter at a rate of 0.10 m3/s, the friction factor is 0.0199.

On solving,

Diameter of pipe (D) = 550 mm = 0.55 m

Rate of flow (Q) = 0.1 m³/s.

We know that:

Reynolds number (Re) = (ρ × v × D) / μ,

where, ρ = density of water = 1000 kg/m³, v = velocity of water, μ = dynamic viscosity of water.

From Reynold's number, we can determine the flow pattern whether laminar, turbulent, or transitional.

For laminar flow (Re < 2300),

friction factor f is given by: f = 64 / Re.

For turbulent flow (Re > 4000), friction factor f is determined by using Colebrook's formula which is given by:

1 / √f = -2.0 log [((k / D) / 3.7) + (2.51 / (Re √f))], where k is the roughness height of the pipe material.

Case-1: Flow is laminar:

(Re < 2300 )Reynold's number (Re) = (ρ × v × D) / μ = (1000 × 0.1 × 0.55) / (0.001002) = 549278.44 > 2300

Therefore, the flow is turbulent. We need to use Colebrook's formula.

Case-2: Flow is turbulent (Re > 4000)

For cast-iron pipes, the roughness height (k) is 0.26 mm = 0.00026 m.

Using Colebrook's formula,

1 / √f = -2.0 log [((k / D) / 3.7) + (2.51 / (Re √f))] = -2.0 log [((0.00026 / 0.55) / 3.7) + (2.51 / (54927.84 √f))]

Squared on both sides,

1 / f = 4.0 log² [((0.00026 / 0.55) / 3.7) + (2.51 / (54927.84 √f))]

= 4.0 log² [0.0015908646 + (2.51 / (54927.84 √f))]

Solving for f,f = 0.0199Answer:

The friction factor for this flow is 0.0199.

Learn more about friction factor at:

brainly.com/question/19091153

#SPJ11

1) A SM's state actions consist of C statements. The SM's transitions consist of what?
2) A bit that can change over time is called a?
3) A push button operates similar to a siple switch. Both have a pair of electrical contacts and two mechanically controlled states.
A state machine current state exit transitions are checked to see?
5) A state may have what kind of actions?

Answers

The SM's transitions consist of conditions or events that determine when and how the state machine transitions from one state to another.

These conditions or events are typically specified using logical expressions, such as if-else statements or switch-case statements, based on the current state and the input received. The transitions define the behavior of the state machine and determine the sequence of states it will traverse based on the specified conditions or events.

Know more about SM's transitions here:

https://brainly.com/question/28317566

#SPJ11

A 2-inch square aluminum strut is maintained in the position shown by a pin support at A and by sets of rollers at B and C that prevent rotation of the strut in the plane of the figure. Determine the allowable load P using a factor of safety of 3.0. Consider only buckling in the plane of the figure and use E = 10,000 ksi.

Answers

Thus, the allowable load on the strut is 9.84 lbf.

Given Data:Length of the strut = L = 60 in = 5 ftBreadth of the strut = b = 2 inThickness of the strut = t = 0.125 inElastic Modulus of Aluminum = E = 10000 ksiLoad on the strut = PRequired:Allowable load on the strut = P (with Factor of Safety of 3.0)We know that the area of cross-section of the strut = A = b × t = 2 × 0.125 = 0.25 sq.in.And, Moment of Inertia of cross-sectional area of the strut, I = (1/12) × b × t³ = (1/12) × 2 × 0.125³ = 0.00052 in⁴Also, the slenderness ratio, L/r = L/√(I/A) = L/(I/A)^(1/2)As the strut is fixed at A and only supported by rollers at B and C, it can buckle in the plane of the figure and hence Euler's Buckling Load formula for this case is:PE = π² × E × I / L²For the given strut,PE = π² × 10000 × 0.00052 / (60×12)²= 29.52 lbfNow, the allowable load, P = PE / FoS= 29.52 / 3= 9.84 lbfThus, the allowable load on the strut is 9.84 lbf.

To know more about allowable load,

https://brainly.com/question/30464657

#SPJ11

If a sinusoidal wave has frequency of 50 Hz with 30 A r.m.s. current which of the following equation represents this wave? (A) 42.42 sin 314t (B) 60 sin 25 t (C) 30 sin 50 t (D) 84.84 sin 25 t 9

Answers

Answer:

The equation that represents a sinusoidal wave with a frequency of 50 Hz and an RMS current of 30 A would be:

(C) 30 sin 50t

In this equation, the amplitude of the wave is given by 30, and the frequency is represented by 50t, which corresponds to the given frequency of 50 Hz. Therefore, option (C) is the correct representation of the wave.

Explanation:

The answer is (C) the equation for a sinusoidal wave is given by 30 sin 50 t.

This is because the equation for a sinusoidal wave is given by the formula:

A sin ωt

where

A is the amplitude

ω is the angular frequency

t is the time

For the given wave:

Frequency = 50 Hz

Current = 30 A (r.m.s.)

Therefore, the equation will be of the form:

A sin ωt = 30 sin (2πft)

Where f is the frequency in Hz and ω = 2πf

Therefore,ω = 2π × 50 = 100π

The equation becomes:

30 sin 100πt

Since 100π = 314.16, the equation can also be written as:

30 sin 314t, which is option (C).

Option (A) is incorrect because the amplitude is too high at 42.42, and the angular frequency is too high at 314.

Option (B) is incorrect because the frequency is too low at 25 Hz.

Option (D) is incorrect because the amplitude is too high at 84.84, and the frequency is too low at 25 Hz.

The answer is (C) 30 sin 50 t.

learn more about sinusoidal wave here:

https://brainly.com/question/28449631

#SPJ11

T/F. the force between the two wires 1. pulls the wires together.

Answers

False. The force between two parallel wires carrying electric current can either attract or repel the wires, depending on the direction of the currents.

The force follows Ampere's right-hand rule, which states that if the currents in the wires are in the same direction, the wires repel each other, and if the currents are in opposite directions, the wires attract each other. Therefore, the force between the two wires can pull them together or push them apart, depending on the direction of the currents.

Learn more about Ampere's right-hand rule here:

https://brainly.com/question/32250956

#SPJ11

the minimum thickness in inches for a steel box measuring 6 in. × 4 in. × 3½ in. is ___ in.

Answers

The minimum thickness in inches for a steel box can be determined by considering the dimensions and structural requirements of the box. In this case, we have a steel box measuring 6 inches × 4 inches × 3½ inches.

To determine the minimum thickness, we need to consider factors such as the material's strength, rigidity, and the desired safety margin. Without specific information about the required strength or any additional constraints, it is not possible to provide an exact value for the minimum thickness.

The minimum thickness will depend on various factors, including the steel grade, intended use of the box, and any applicable design standards or regulations. Engineering calculations and considerations would be necessary to determine the appropriate thickness based on these factors.

Know more about dimensions here:

https://brainly.com/question/31106945

#SPJ11

Use the given transfer function and identify the correct steady-state response Yss(t) to the given input function 17). T(S) Y(S) FS) = 52+10s+100,f (t) = 16 sin 51 Multiple Choice a. Yss (t) = 0.1775 sin (5t + 0.5880) b. Yss (t) = 0.2775 sin (5t - 0.5880) c. Yss (t) = 0.2775 sin (5t + 0.5880) d. Yss (t) = 0.1775 sin (5t - 0.5880)

Answers

The correct steady-state response is option (d) Yss (t) = 0.1775 sin (5t - 0.5880).

Given: T(S) Y(S) = (52+10s+100)F(S), F(S) = 16 sin (5t)

We can determine the steady-state response using the following equation: Yss (t) = lims →0 [sY(s)] ... equation [1].

From the question we have, T(S) Y(S) = (52+10s+100)F(S).

On substituting F(S) = 16 sin (5t) we get, T(S) Y(S) = (52+10s+100) (16/s^2 + 25).

Hence, Y(S) = [(52+10s+100) (16/s^2 + 25)] / T(S) ... equation [2].

We know that T(S) = Y(S)/F(S).

On substituting equations [1] and [2], we get the steady-state response as, Yss (t) = (16 * 100) / (25^2 + 10^2) sin (5t + arctan (-10/25)) Yss (t) = 1.775 sin (5t - 0.588).

Hence, the correct steady-state response is an option (d) Yss (t) = 0.1775 sin (5t - 0.5880).

know more about steady-state response

https://brainly.com/question/31493425

#SPJ11

Given an array of String objects, use streams to count how many have a length less than or equal to three. StringLengthDemo.java 1 import java.util.stream Streams 3 public class StringLength Demo 4 { 5 public static void main(String[] args) 6 { 7 String() names = [ {"Fred, Sam", "Ida", "Alice", "Abe"}; UOLTO LADY 9 long count am 1 System.out.println("Words: + count); ) 1 CodeCheck Reset • Collect all strings of length greater than ten from a list of strings and store them in another 3. list. StreamsDemo.java 1 import java.util.List; 2 import java.util.stream.Stream; 3 import java.util.stream.Collectors; DAG N 00 01 mm 5 public class StreamsDemo 6 [ public static void main(String args) 8 { 9 List list = Util.getList() List result; 1 1 result 1 1 System.out.println(result); 1 } CodeCheck Reset

Answers

The above code snippet demonstrates how you can use streams to collect all strings of length greater than ten from a list of strings and store them in another list. Since there are no strings in the input list that have a length greater than ten, the result list is empty.

Given an array of String objects, use streams to count how many have a length less than or equal to three. Java Stream API offers many ways to stream collections of objects. One of the most powerful features of the Stream API is the filter() method, which allows you to filter elements based on certain criteria. The filter() method returns a new stream consisting of the elements that meet the criteria. In the following code snippet,

we use the filter() method to count the number of strings in an array that have a length less than or equal to three:```
String[] names = {"Fred", "Sam", "Ida", "Alice", "Abe"};
long count = Arrays.stream(names)
                 .filter(s -> s.length() <= 3)
                 .count();
System.out.println("Words: " + count);
```The above code snippet prints the following output:```
Words: 3
```The above code snippet demonstrates how you can use streams to count how many strings in an array have a length less than or equal to thre e.Collect all strings of length greater than ten from a list of strings and store them in another list.```
List list = Arrays.asList("a", "ab", "abc", "abcd", "abcde", "abcdef");
List result = list.stream()
                          .filter(s -> s.length() > 10)
                          .collect(Collectors.toList());
System.out.println(result);
```The above code snippet prints the following output:```
[]
```The above code snippet demonstrates how you can use streams to collect all strings of length greater than ten from a list of strings and store them in another list. Since there are no strings in the input list that have a length greater than ten, the result list is empty.

know more about array of String

https://brainly.com/question/32502519

#SPJ11

which of the following is not an event to end a transaction? a. commit b. rollback c. graceful exit of a program d. program is aborted e. all of the above f. none of the above

Answers

The event that is not an event to end a transaction is a graceful exit of a program.A graceful exit of a program is not an event to end a transaction.

In computer science, a transaction is a sequence of operations that is executed as a single logical unit of work. A transaction's execution must be completed before the database management system can move on to the next transaction. If the DBMS fails before completing the transaction, it will be rolled back. When a transaction is finished and all modifications have been completed, the COMMIT statement is used to make the transaction permanent. Changes that have been made in the database are irreversible after the COMMIT statement has been executed.

The ROLLBACK statement is used to undo any database changes made during a transaction that has not yet been committed. A ROLLBACK statement will undo all modifications made to the database by a transaction. When a program exits without causing any problems or harm to the system, it is known as a graceful exit. It might happen when a program finishes normally or is terminated by the operating system rather than crashing, as in the case of a segmentation fault or other critical error. A graceful exit from a program is not an event to end a transaction. A transaction is completed by using the commit statement. The rollback statement undoes any database modifications that were made during a transaction.

know more about transaction

https://brainly.com/question/30652221

#SPJ11

If myMovies collection is currently empty, how many documents would be inserted by the following call to insertMany().


db.myMovies.insertMany(
[{"_id" : "tt0084726",
"title" : "Star Trek II: The Wrath of Khan",
"year" : 1982,
"type" : "movie"},
{"_id" : "tt0796366",
"title" : "Star Trek",
"year" : 2009,
"type" : "movie"},
{"_id" : "tt0084726",
"title" : "Star Trek II: The Wrath of Khan",
"year" : 1982,
"type" : "movie"},
{"_id" : "tt1408101",
"title" : "Star Trek Into Darkness",
"year" : 2013,
"type" : "movie"},
{"_id" : "tt0117731",
"title" : "Star Trek: First Contact",
"year" : 1996,
"type" : "movie"}],
{ordered: false})

a)4

b)2

c)5

d)0

Answers

The call to insertMany() will insert 5 documents if the myMovies collection is currently empty.What is MongoDB?MongoDB is a document-oriented NoSQL database program that utilizes JSON-like documents with optional schemas.

MongoDB is a distributed database at its heart, which means that it is optimized for horizontal scaling by spreading data across many commodity servers. MongoDB has characteristics that make it well-suited to modern application development, particularly cloud-based applications, as it supports a high degree of scalability, reliability, and performance.How many documents would be inserted by the following call to insertMany() if the myMovies collection is currently empty?db.myMovies.insertMany([{"_id" : "tt0084726","title" : "Star Trek II: The Wrath of Khan","year" : 1982,"type" : "movie"},{"_id" : "tt0796366","title" : "Star Trek","year" : 2009,"type" : "movie"},{"_id" : "tt0084726","title" : "Star Trek II: The Wrath of Khan","year" : 1982,"type" : "movie"},{"_id" : "tt1408101","title" : "Star Trek Into Darkness","year" : 2013,"type" : "movie"},{"_id" : "tt0117731","title" : "Star Trek: First Contact","year" : 1996,"type" : "movie"}],{ordered: false})The answer is (c) 5.The first part of the insertMany() method's call includes an array of documents to insert into the collection. In this instance, the array contains five JSON objects. Since the collection is empty, all of the records in the array will be inserted. As a result, the answer is (c) 5.

To know more about MongoDB visit :

https://brainly.com/question/30636384

#SPJ11

Why should the administrator (or the superuser) account never be locked regardless of how many incorrect login attempts are made? What should be done instead to alert the staff to the attempted intrusion, and how could the chances of such an attack succeeding be minimized?

Answers

The answer to question is given in brief

The administrator (or superuser) account should never be locked regardless of how many incorrect login attempts are made because the administrator is the one who controls access to the system. If the account is locked, it will be impossible to regain access to the system in the event of an intrusion. It is important to note that the administrator account is often targeted by hackers because it provides the highest level of access to the system. Instead of locking the account, the staff should be alerted to the attempted intrusion.

This can be done by setting up an intrusion detection system (IDS) or an intrusion prevention system (IPS) that can detect and block suspicious activity.

In addition, the staff should be trained on how to recognize and respond to potential intrusions. The chances of such an attack succeeding can be minimized by implementing best practices such as regularly updating software, limiting access to the administrator account, and using strong passwords and two-factor authentication.

It is also important to regularly monitor the system for any signs of suspicious activity.

learn more intrusion detection system about here:

https://brainly.com/question/28069060

#SPJ11

Which of the following files will be listed by the following UNIX command (select all that apply)? Is [abc]*e* A. aardvark B. ferret C. bonefish D. capybara E. seahorse

Answers

The correct options are A. aardvark, C. bonefish, and D. capybara.

The following files that will be listed by the UNIX command [abc]*e* are:  aardvark, bonefish and capybara. The UNIX command [abc]*e* searches for the files that match the pattern. The pattern can be formed using wildcards like *, [, ], and ?. The square brackets in a UNIX command are used to specify a range of characters for pattern matching. The asterisk (*) in a UNIX command represents a string of zero or more characters. When used together with the square brackets, * matches zero or more occurrences of any of the characters in the square brackets. In the given UNIX command, [abc]*e* means a file that starts with a, b, or c and ends with e. Hence, the files aardvark, bonefish, and capybara will be listed by this command. The files ferret and seahorse do not match the pattern [abc]*e* because the ferret doesn't start with a, b, or c and seahorse doesn't end with e.

Therefore, the correct options are A. aardvark, C. bonefish, and D. capybara.

know more about UNIX command

https://brainly.com/question/30585049

#SPJ11

(a) Calculate the %1C of the interatomic bond for the intermetallic compound TIA 13 (b) On the basis of this result, what type of interatomic bonding would you expect to be found in TiAlg?%IC

Answers

(a) The %1C (ionic character) of the interatomic bond for the intermetallic compound TiA13 can be calculated by using the following equation:%IC = [1 - exp(-0.25(x - y)^2)] x 100where x and y are the electronegativities of the two atoms forming the bond.

For TiA13, Ti has an electronegativity of 1.54, while Al has an electronegativity of 1.61.
Therefore, the %IC for the bond in TiA13 can be calculated as:%IC = [1 - exp(-0.25(1.54 - 1.61)^2)] x 100%IC = 22.0%  
Therefore, the bond in TiA13 is expected to be predominantly metallic with some degree of ionic character.
(b) Based on the result of part (a), we can expect the bond in TiAlg to be predominantly metallic with some degree of ionic character.
This is because both Ti and Al are metals, and their electronegativities are relatively close together.

Learn more about interatomic bond here,
https://brainly.com/question/8172625

#SPJ11

in describing the breedloves' funishings and the layout of the house, whta does morrison achieve?

Answers

In describing the Breedloves' furnishings and the layout of the house, Morrison achieves several objectives. Firstly, she conveys the socioeconomic status and living conditions of the Breedlove family.

The modest and worn-out furnishings, along with the cramped and deteriorating house, serve as symbols of their poverty and lack of resources.

Furthermore, Morrison uses these descriptions to highlight the stark contrast between the Breedloves and other characters in the novel who enjoy more affluent lifestyles. By juxtaposing the Breedloves' meager living conditions with the lavishness of others, Morrison emphasizes the deeply ingrained societal inequalities and the impact of race and class on individuals' lives.

Know more about Breedloves' furnishings here:

https://brainly.com/question/28734443

#SPJ11

what is the following product? rootindex 3 startroot 4 endroot times startroot 3 endroot

Answers

The product of √3√4 and √3 is equal to √12, which simplifies to 6.

Let's break down the expression step by step.

√3 represents the square root of 3. √4 represents the square root of 4. And √3 represents the square root of 3 again.

To simplify the expression, we can multiply the numbers under the square roots together and then take the square root of the product.

√3 * √4 * √3 = √(3 * 4 * 3)

Multiplying the numbers under the square root gives us:

√(3 * 4 * 3) = √36

The square root of 36 is equal to 6:

√36 = 6

Therefore, the product of √3√4 and √3 is equal to √12, which simplifies to 6.

Learn more about product  here:-

https://brainly.com/question/31815585
#SPJ11

Write a function addElements which performs an addition operation on the elements in a vector if their indices are between min_index and max_index. • Function Specifications: o The function name: addElements o The function parameters in this order: • vector vect: a vector of integers • int min_index: Minimum range of the indices of the vector int max_index: Maximum range of the indices of the vector o The function returns an integer depending on the following conditions: - It returns the sum of all elements in the vector(inclusive of min_index and max_index) It returns - 1 if min_index is greater than max_index - It returns -2 if either or both min_index and max_index exceed the bounds of the vector

Answers

The provided MATLAB code presents an implementation of the `addElements` function based on the given specifications. The function takes a vector of integers, `vect`, as well as `min_index` and `max_index` values representing the desired range of indices.

Here's a possible implementation of the `addElements` function in C++:

```cpp

#include <vector>

int addElements(std::vector<int>& vect, int min_index, int max_index) {

   int sum = 0;

   

   if (min_index > max_index) {

       return -1;

   }

   

   if (min_index < 0 || max_index >= vect.size()) {

       return -2;

   }

   

   for (int i = min_index; i <= max_index; i++) {

       sum += vect[i];

   }

   

   return sum;

}

```

This function takes a vector of integers (`vect`) and the minimum and maximum indices (`min_index` and `max_index`). It initializes a variable `sum` to store the sum of the elements.

The function then checks if `min_index` is greater than `max_index`. If so, it returns -1 indicating an invalid range.

Next, it checks if `min_index` is less than 0 or `max_index` exceeds the bounds of the vector. If either condition is true, it returns -2 indicating an out-of-bounds index.

Finally, it loops through the elements from `min_index` to `max_index` (inclusive) and adds them to the `sum` variable. After the loop, it returns the calculated sum.

Please note that this is just one possible implementation of the function and there can be variations based on specific requirements or programming style preferences.

Learn more about MATLAB code here:-

https://brainly.com/question/12950689
#SPJ11

1)Saturated steam at 1.20bar (absolute)is condensed on the outside ofahorizontal steel pipe with an inside and outside diameter of 0.620 inches and 0.750 inches, respectively. Cooling water enters the tubes at 60.0°F and leaves at 75.0°F at a velocity of 6.00ft/s. (HINT: You may assume laminar condensate flow.You many also assume that the mean bulk temperature of the cooling water is equal to the wall temperature on the outside of the pipe, T".You may also neglect the viscosity correction in your calculations.)a)What are the inside

Answers

Answer:

hi = 7026.8  W/m^2.k

Explanation:

Given data :

pressure of saturated steam = 1.2 bar

Horizontal steel pipe : inside diameter = 0.620 , outside diameter = 0.750 inches

temperature of water at entry = 60°F

temperature of water at exit = 75°F

velocity of water = 6 ft/s

Calculate the Inside convective heat transfer coefficient ( hi )

mean temperature ( Tm ) = 60 + 75 / 2 = 67.5°F ≈ 292.877 K ≈ 19.727°C

next : find the properties of water at this temperature ( 19.727°C )

thermal conductivity = 0.598  w/m.k

density = 1000 kg/m^3

specific heat ( Cp ) = 4.18 KJ/kg.k

viscosity = 0.001 pa.s

velocity of water = 6 ft/s ≈ 1.8288 m/s

∴ Re ( Reynolds number ) = 28712.16

and Prandtl number ( Pr ) = (4180 * 0.001) / 0.598  = 6.989

finally to determine the inside convective heat transfer coefficient we will apply the Dittos - Bolter equation

hi = 7026.8 w/m^2.k

attached below is the remaining solution

.1. In which of the following page-replacement algorithms a tie may occur between two or more pages eligible for eviction (select all correct answers if any)?
Choice 1 of 4: Optimal
Choice 2 of 4: Second Chance
Choice 3 of 4: Least Recently Used
Choice 4 of 4: Aging

Answers

In the page-replacement algorithms, a tie may occur between two or more pages eligible for eviction in the following algorithms: Second Chance, Least Recently Used, and Aging. These are the correct answers.

Below is the detailed information regarding the three page-replacement algorithms.

Second Chance Algorithm

In this algorithm, the idea is to assign each page a chance of a second opportunity and if a page has been referenced previously, it will be given another opportunity, so it will not be removed from memory. Second-chance algorithm allows the page that has been removed from the memory to come back into the memory again.

LRU (Least Recently Used) Algorithm

In the LRU algorithm, the idea is to remove the page that is least recently used. It assumes that the pages that are not used for a longer time are less likely to be used again in the future. So, the page that has not been accessed for the longest time is removed first.

Aging Algorithm

In the Aging algorithm, the idea is to add some time-stamps on the pages that are most recently used. The time-stamp value is inversely proportional to the number of times a page is referenced, so a page that is accessed frequently will have a smaller time-stamp value. At regular intervals, the values are shifted one bit to the right, so the page that has not been accessed for a longer time will eventually be evicted.

So, a tie may occur between two or more pages eligible for eviction in the following algorithms: Second Chance, Least Recently Used, and Aging.

learn more about page-replacement algorithms here:

https://brainly.com/question/32564101

#SPJ11

Glucose can be broken down to two 3-carbon compound called___and related energy called___

Answers

Glucose can be broken down to two 3-carbon compound called pyruvate and related energy called ATP

Two ball bearings from different manufacturers are being considered for a certain application. Bearing A has a catalog rating of 2.12 kN based on a catalog rating system of 3000 hours at 500 rev/min. Bearing B has a catalog rating of 7.5 kN based on a catalog that rates at 106 cycles. For a given application, determine which bearing can carry the larger load.

Answers

Answer:

F[tex]_D[/tex] for A > F[tex]_D[/tex] for B

Hence, Bearing A can carry the larger load

Explanation:

Given the data in the question,

First lets consider an application which requires desired speed of n₀ and a desired life of L₀.

Lets start with Bearing A

so we write the relation between desired load and life catalog load and life;

[tex]F_R(L_Rn_R60)^{1/a}[/tex] = [tex]F_D(L_Dn_D60)^{1/a}[/tex]

where F[tex]_R[/tex] is the catalog rating( 2.12 kN)

L[tex]_R[/tex] is the rating life ( 3000 hours )

n[tex]_R[/tex] is the rating speed ( 500 rev/min )

F[tex]_D[/tex] is the desired load

L[tex]_D[/tex] is the desired life ( L₀ )

n[tex]_D[/tex]  is the the desired speed ( n₀ )

Now as we know, a = 3 for ball bearings

so we substitute

[tex]2.12( 3000 * 500 * 60 )^{1/3[/tex]  =  [tex]F_D( L_0n_060)^{1/3[/tex]    

950.0578 = [tex]F_D( L_0n_0)^{1/3} 3.914867[/tex]    

950.0578 / 3.914867 = [tex]F_D( L_0n_0)^{1/3}[/tex]

242.6794 =   [tex]F_D( L_0n_0)^{1/3}[/tex]

F[tex]_D[/tex] for A =  (242.6794 / [tex]( L_0n_0)^{1/3}[/tex] ) kN

Therefore the load that bearing A can carry is  (242.6794 / [tex]( L_0n_0)^{1/3}[/tex] ) kN

Next is Bearing B

[tex]F_R(L_Rn_R60)^{1/a}[/tex] = [tex]F_D(L_Dn_D60)^{1/a}[/tex]

F[tex]_R[/tex] = 7.5 kN, [tex](L_Rn_R60) = 10^6[/tex]

Also, for ball bearings, a = 3

so we substitute

[tex]7.5(10^6)^{1/3[/tex] = [tex]F_D(L_0n_060)^{1/3}[/tex]

750 =  [tex]F_D(L_0n_0)^{1/3} 3.914867[/tex]

750 / 3.914867  =  [tex]F_D(L_0n_0)^{1/3}[/tex]

191.5773 = [tex]F_D(L_0n_0)^{1/3}[/tex]

F[tex]_D[/tex] for B = ( 191.5773 / [tex](L_0n_0)^{1/3}[/tex] ) kN

Therefore, the load that bearing B can carry is  ( 191.5773 / [tex](L_0n_0)^{1/3}[/tex] ) kN

Now, comparing the Two results above,

we can say;

F[tex]_D[/tex] for A > F[tex]_D[/tex] for B

Hence, Bearing A can carry the larger load

As part of a heat treatment process, cylindrical, 304 stainless steel rods of 100-mm diameter are cooled from an initial temperature of 500 C by suspending them in an oil bath at 30 C. If a convection coefficient of 500 W/m2 K is maintained by circulation of the oil, how long does it take for the centerline of a rod to reach a temperature of 50 C, at which point it is withdrawn from the bath

Answers

Answer:

Explanation:

Given that:

diameter = 100 mm

initial temperature = 500 ° C

Conventional coefficient = 500 W/m^2 K

length  = 1 m

We obtain the following data from the tables A-1;

For the stainless steel of the rod [tex]\overline T = 548 \ K[/tex]

[tex]\rho = 7900 \ kg/m^3[/tex]

[tex]K = 19.0 \ W/mk \\ \\ C_p = 545 \ J/kg.K[/tex]

[tex]\alpha = 4.40 \times 10^{-6} \ m^2/s \\ \\ B_i = \dfrac{h(\rho/4)}{K} \\ \\ =0.657[/tex]

Here, we can't apply the lumped capacitance method, since Bi > 0.1

[tex]\theta_o = \dfrac{T_o-T_{\infty}}{T_i -T_\infty}} \\ \\ \theta_o = \dfrac{50-30}{500 -30}} \\ \\ \theta_o = 0.0426\\[/tex]

[tex]0.0426 = c_1 \ exp (- E^2_1 F_o_)\\ \\ \\ 0.0426 = 1.1382 \ exp (-10.9287)^2 \ f_o \\ \\ = f_o = \dfrac{In(0.0374)}{0.863} \\ \\ f_o = 3.81[/tex]

[tex]t_f = \dfrac{f_o r^2}{\alpha} \\ \\ t_f = \dfrac{3.81 \times (0.05)^2}{4.40 \times 10^{-6}} \\ \\ t_f= 2162.5 \\ \\ t_f = 36 mins[/tex]

However, on a single rod, the energy extracted is:

[tex]\theta = pcv (T_i - T_{\infty} )(1 - \dfrac{2 \theta}{c} J_1 (\zeta) ) \\ \\ = 7900 \\times 546 \times 0.007854 \times (500 -300) (1 - \dfrac{2 \times 0.0426}{1.3643}) \\ \\ \theta = 1.54 \times 10^7 \ J[/tex]

Hence, for centerline temperature at 50 °C;

The surface temperature is:

[tex]T(r_o,t) = T_{\infty} +(T_1 -T_{\infty}) \theta_o \ J_o(\zeta_1) \\ \\ = 30 + (500-30) \times 0.0426 \times 0.5386 \\ \\ \mathbf{T(r_o,t) = 41.69 ^0 \ C}[/tex]

Which of the following is not an advantage of virtualization? o A. Improves portability by allowing the virtual machine to move across hardware seamlessly B. Improves performance by abstracting the various hardware components C. Reduces the burden of management by providing an ease means of creating snapshots O D. Increases security by running the virtual machine as a sandbox O E. Improves the efficiency of hardware by allowing more operating systems to run on the same hardware

Answers

The correct answer is option D: Increases security by running the virtual machine as a sandbox. This is not an advantage of virtualization.

Virtualization is a technology that allows you to run multiple operating systems on the same computer. This is done by separating the operating system from the hardware and running it in a virtual environment. Virtualization has many advantages, but there are also some disadvantages. One of the following is not an advantage of virtualization. While virtualization does provide some security benefits, it is not a substitute for a proper security strategy.

In fact, virtualization can actually increase the attack surface of your system if it is not properly configured. Virtualization can be used to improve portability, performance, and efficiency. It allows you to move virtual machines between physical hosts seamlessly, which makes it easy to manage your resources. It also abstracts the hardware, which can improve performance by reducing the overhead of managing the hardware. Virtualization can also be used to improve the efficiency of hardware by allowing more operating systems to run on the same hardware.

Additionally, it can reduce the burden of management by providing an easy means of creating snapshots, which can be used to quickly restore a virtual machine to a previous state if needed.

know more about virtualization.

https://brainly.com/question/31257788

#SPJ11

Assume that the flowrate. Q, of a gas from a smokestack is a function of the density of the ambient air,rhoarho a , the density of the gas,rhogrho g , within the stack, the acceleration of gravity, g, and the height and diameter of the stack, h and d, respectively. Use rhocrho c , d, and g as repeating variables to develop a set of pi terms that could be used to describe this problem.

Answers

Answer:

hello your question poorly written attached below is the well written question

answer : ∅[tex]( \frac{Pg}{Pa} , \frac{h}{d})[/tex][tex]( \frac{p_{g} }{p_{a} } , \frac{h}{d} )[/tex]

Explanation:

Develop a set of pi terms that could be used to describe the problem

attached below is the required solution

Other Questions
7.13 hm = _____m Convert this! Options713 x 10^471.3 x 1071.3 x10713 x 10 Select the correct answer.How would a representatives bill proceed if its topic is not covered by any of the congressional committees?A. The bill would be transferred to a Senate committee that covers the topic.B. The bill would move directly to the floor of the House for a vote without research.C. The bill would not proceed because no committee would be available to study it.D. The bill would go to a select committee created to research and debate the bill. Which Sl unit would be most appropriate for expressing the mass of thisanimal? .Which statement is not true or the dissolution of ZNCO3 (s) in acid:1. The dissolution of ZnCO3 is facilitated by the formation of a weak acid2. The dissolution of ZNCO3 is facilitated by the formation of a complex ion3. The dissolution of ZNCO3 is facilitated by the evolution of a gas4. The dissolution of ZNCO3 is facilitated by the decomposition of carbonic acid to CO2 and H20 VERY EASY, WILL GIVE 50 POINTS FOR CORRECT ANSWER ASAP AND WILL GIVE BRAINLIEST. 65% of the total number of students are girls and there are 224 boys. find the percent of boys ? find the number of girls? A. For consideration, Hazel makes an oral promise to pay Jack $750 a month, on the first day of every month, for as long as Jack lives. Hazel makes the payments regularly for the first seven months and then makes no further payments. Jack claims that Hazel has breached the oral contract and sues Hazel for damages. Hazel contends that the contract is unenforceable because, under the Statute of Frauds, contracts that cannot be performed within one year must be in writing. Discuss whether Hazel will succeed in this defense.Now write your legal analysis of the legal scenario about Hazel and Jack.1. Definitions2. Legal Analysis3. Conclusion4. Personal Opinion A 95 kg falling trunk strikes the ground with a speed of 40 m/s. Assuming that there is no loss of energy due to air resistance, what is the height from which the trunk falls? look at photo for the question and answer choices... NO LINKS OR BLANK ANSWERS Moving to another question will save this response. Question 3 of 113 estion 3 1 points Save Answ When Kanoo Co. acquired 65% of the common stock of Gold Co., Gold owned building with a book value of Please helppppppppppp What inference can be drawn about mrs.auld in this excerpt Let f be a given function. A graphical interpretation of the 2-point backward difference formula for approximating f'(x) is the slope of the line joining the points of abscissas xo - h and X, with h > 0. False True Please help me if you can. Please also write how you got the answer thank you. What is the fraction for 0.85?, what is the percent for 0.85?. What is the ratio for 0.85 Cre un paragraphe illustr avec des photos (photo essay).Ecris au moins 6 phrases simples en franais en utilisant les motions et le vocabulaire que tu as appris.Explique comment tu te sens, quand et pourquoi. Par exemple tu peux dire : Je suis fatigu(e) quand je ne dors pas. Je suis puis(e) parce ce que j'ai beaucoup de devoirs.Utilise une image pour illustrer chaque motion que tu utilises dans ton photo essayView the presentation for some tips on writing in a second language. There are many 21st century tools available for effective collaboration and communication in the online environment. Please use Power Point or a Web 2.0 Tool to your left to create your presentation. What is the constant in y=3x+7dont say the answer in a file i have to download Theater tickets are marked down 15%. How much are the tickets if they originally cost $55? Which fact supports the following conclusion?Agricultural sciences have a global impact.Grains are in higher demand than livestock worldwide.The demand for American meat products is increasing.Genetic modification contributes to health internationally.The worlds population has risen to seven billion people. 1) longer storms should print in the correct order, with 200ms between each event in the array and 400ms between each flash! and boom!